www.vorkurse.de
Ein Projekt von vorhilfe.de
Die Online-Kurse der Vorhilfe

E-Learning leicht gemacht.
Hallo Gast!einloggen | registrieren ]
Startseite · Mitglieder · Teams · Forum · Wissen · Kurse · Impressum
Forenbaum
^ Forenbaum
Status Mathe-Vorkurse
  Status Organisatorisches
  Status Schule
    Status Wiederholung Algebra
    Status Einführung Analysis
    Status Einführung Analytisc
    Status VK 21: Mathematik 6.
    Status VK 37: Kurvendiskussionen
    Status VK Abivorbereitungen
  Status Universität
    Status Lerngruppe LinAlg
    Status VK 13 Analysis I FH
    Status Algebra 2006
    Status VK 22: Algebra 2007
    Status GruMiHH 06
    Status VK 58: Algebra 1
    Status VK 59: Lineare Algebra
    Status VK 60: Analysis
    Status Wahrscheinlichkeitst

Gezeigt werden alle Foren bis zur Tiefe 2

Navigation
 Startseite...
 Neuerdings beta neu
 Forum...
 vorwissen...
 vorkurse...
 Werkzeuge...
 Nachhilfevermittlung beta...
 Online-Spiele beta
 Suchen
 Verein...
 Impressum
Das Projekt
Server und Internetanbindung werden durch Spenden finanziert.
Organisiert wird das Projekt von unserem Koordinatorenteam.
Hunderte Mitglieder helfen ehrenamtlich in unseren moderierten Foren.
Anbieter der Seite ist der gemeinnützige Verein "Vorhilfe.de e.V.".
Partnerseiten
Weitere Fächer:

Open Source FunktionenplotterFunkyPlot: Kostenloser und quelloffener Funktionenplotter für Linux und andere Betriebssysteme
Forum "Mathematik-Wettbewerbe" - Neue Aufgaben Nr. 10
Neue Aufgaben Nr. 10 < Wettbewerbe < Schule < Mathe < Vorhilfe
Ansicht: [ geschachtelt ] | ^ Forum "Mathematik-Wettbewerbe"  | ^^ Alle Foren  | ^ Forenbaum  | Materialien

Neue Aufgaben Nr. 10: Übungsaufgabe
Status: (Übungsaufgabe) Übungsaufgabe Status 
Datum: 15:37 Sa 19.02.2005
Autor: Hanno

Hallo an alle!

Quelle: Kanadische Mathematik Olympiade 1971

Seien $x,y$ positive, reelle Zahlen mit $x+y=1$. Beweise, dass dann
[mm] $\left(1+\frac{1}{x}\right) \left(1+\frac{1}{y}\right) \geq [/mm] 9$
gilt.


Liebe Grüße,
Hanno

        
Bezug
Neue Aufgaben Nr. 10: Frage (beantwortet)
Status: (Frage) beantwortet Status 
Datum: 23:15 Sa 19.02.2005
Autor: Teletubyyy

Hi Hanno,

meine Lösung ist vielleicht unelegant und simple rechnerei - aber klappt.

Nachaufgabenstellung ist y=1-x. Als Lösung reicht es nun vollkommen aus, die Funktion [mm]f(x)=\left(1+ \frac{1}{x}\right) \left(1+\frac{1}{1-x}\left)=\frac{-x^2+x+1}{x-x^2}[/mm] zu diskutieren und erhällt für das Intervall (0;1) zwischen den Definitionslücken den Tiefpunkt (1.5|9)!!! q.e.d

Gruß Samuel

Bezug
                
Bezug
Neue Aufgaben Nr. 10: Symmetrie nutzen!
Status: (Mitteilung) Reaktion unnötig Status 
Datum: 07:59 So 20.02.2005
Autor: Peter_Pein

Klasse, Samuel!

Bis auf den Tippfehler am Schluss war das auch meine Idee. Da ich aber faul bin und nicht gerne er- ähh - gebrochen rationale Funktionen ableite, argumentierte ich mit der offensichtlichen Symmetrie von $ [mm] f(x)=\left(1+ \frac{1}{x}\right) \left(1+\frac{1}{1-x}\right)$ [/mm] um [mm] $x_{0}=\frac{1}{2}$. [/mm] Aus ihr folgt, dass f dort ein Extremum haben muss oder konstant ist. Das letzte geht schon wegen der Pole bei 0 und 1 nicht. Dort geht f gegen [mm] $+\infty$, [/mm] also ist bei [mm] $x_{0}$ [/mm] ein Minimum. Einsetzen. Fertig.

Schönes Wochenende euch allen,
Peter

Bezug
                
Bezug
Neue Aufgaben Nr. 10: Antwort
Status: (Antwort) fertig Status 
Datum: 09:11 So 20.02.2005
Autor: Hanno

Hallo Samuel!

Deine LÖsung ist korrekt, ich hätte es allerdings so gemacht:
Ausklammern führt zu [mm] $1+\frac{1}{x}+\frac{1}{y}+\frac{1}{xy}=1+\frac{x+y+1}{xy}=1+2\frac{xy}$, [/mm] also zur äquivalenten Ungleichung [mm] $xy\leq\frac{1}{4}$. [/mm]
Die folgt aber direkt aus der Voraussetzung, da [mm] $\sqrt{xy}\leq\frac{x+y}{2}=\frac{1}{2}\Rightarrow xy\leq\frac{1}{4}$ [/mm] gilt.

Liebe Grüße,
Hanno

Bezug
Ansicht: [ geschachtelt ] | ^ Forum "Mathematik-Wettbewerbe"  | ^^ Alle Foren  | ^ Forenbaum  | Materialien


^ Seitenanfang ^
www.vorkurse.de
[ Startseite | Mitglieder | Teams | Forum | Wissen | Kurse | Impressum ]